Which one of the following, if substituted for the constraint that Fetter's essay cannot have the same type of theme ...

katia on June 10, 2018

How to substitute conditions

I'm having trouble understanding how to get to answer choice B, and overall how to figure out the relative 'effect' that conditions have on determining assignments. I understand that this is related to guru games, but how do you evaluate the effect of a condition?

Reply
Create a free account to read and take part in forum discussions.

Already have an account? log in

Christopher on June 10, 2018

@katia

These questions are typically (if not always) near the end of the game, which gives you the advantage of having several hypotheticals written out (if you've written them out) and a decent understanding of how the rules interact. With some of these questions, it's easiest to compare the potential rules with hypotheticals that you've already written out or to make sure that the rules interact the same way. In this case, it has to do mostly with rule interaction.

The basic rules mean the game is set up like this:

K/H _ G _ _
R _ _ _ _

Most of this is fairly straightforward. K/H in 1 because 1 being Rural (rule 1) eliminates J (rule 5), G being 3 eliminates G (rule 4), and K being immediately before F eliminates F (rule 2). G is 3 because of rule 4. R is 1 because of rule 1. However, you can make a couple further deductions from these rules (which will hopefully have become clear in the hypotheticals even if you don't see it right off the bat), and those are that 2 must be urban and that 3 must be rural.

Rule 2 (K is adjacent to F) means that [KF] must be either [1,2] or [4,5].

As [1,2] rule three requires that F be U. So 2 will be U if [KF] are [1,2]

As [4,5] there is no determination of whether K or F is Urban or Rural, but rule 3 requires that one of them must be each. Since J must be U, that means that there will always be a U in 2.

Since there must be both a rural AND an urban AFTER 3G and BEFORE 3G (either J or [KF]), then 3G must be Rural.

Notice that most of this hinges on Rule 3. If [KF] could both be rural, then none of this can be deduced, so any rule that replaces Rule 3 must have the same effect, which is to say 2 is U and 3 is R.

(A) partially does this, but by making only H rural, G could be urban, which could allow [KF] to be both rural.

(B) ensuring that G is rural, maintains the 1 urban before and 1 urban after effect of Rule 3. Ensuring that H is rural precludes the possibility of H and J both being urban and K and F being both rural. This is the right answer.

(C) this is contrary to the current rule. Any hypothetical in which [KF] are 1 and 2, F and J will both be urban.

(D) doesn't limit G to being rural, which is a key feature of Rule 3.

(E) has the opposite effect of Rule 3.

Does that help?